310 lines
12 KiB
TeX
310 lines
12 KiB
TeX
\documentclass[a4paper,10pt, table]{/media/documents/Cours/Prof/Enseignements/2014-2015/Archive/2014-2015/tools/style/classDS}
|
|
\usepackage{/media/documents/Cours/Prof/Enseignements/2014-2015/Archive/2014-2015/2014_2015}
|
|
|
|
% Title Page
|
|
\titre{DST 2}
|
|
% \seconde \premiereS \PSTMG \TSTMG
|
|
\classe{\TSTMG}
|
|
\date{Mercredi 18 mars 2015}
|
|
%\duree{1 heure}
|
|
%\sujet{%{{infos.subj%}}}
|
|
% DS DSCorr DM DMCorr Corr
|
|
\typedoc{DS}
|
|
|
|
\printanswers
|
|
|
|
\begin{document}
|
|
\maketitle
|
|
|
|
Le barème est donné à titre indicatif, il pourra être modifié.
|
|
|
|
\begin{questions}
|
|
|
|
\question[9]
|
|
% Centre étranger juin 2014
|
|
|
|
\emph{Les deux parties de l'exercice peuvent être traitées de manière indépendante}
|
|
|
|
L'entreprise SAPIQ commercialise des pots de moutarde de 800~g. Un pot est déclaré \textbf{conforme} s'il contient entre 790~g et 810~g de moutarde.
|
|
|
|
\medskip
|
|
|
|
\textbf{Partie A}
|
|
\medskip
|
|
|
|
L'entreprise dispose de deux machines $m_{1}$ et $m_{2}$.
|
|
|
|
La première machine $m_{1}$ produit 60\,\% des pots fabriqués par l'entreprise, le reste de la fabrication étant assuré par la machine $m_{2}$.
|
|
|
|
7\,\% des pots produits par la machine $m_{1}$ sont non conformes, alors que la proportion de pots non conformes produits par la la machine $m_{2}$ est de 2\,\% seulement.
|
|
|
|
On prélève un pot au hasard dans la production totale.
|
|
|
|
On adopte les notations suivantes :
|
|
|
|
\setlength\parindent{6mm}
|
|
\begin{itemize}
|
|
\item[$\bullet~~$] $M_{1}$ désigne l'évènement \og le pot provient de la machine $m_{1}$. \fg
|
|
\item[$\bullet~~$] $M_{2}$ désigne l'évènement \og le pot provient de la machine $m_{2}$. \fg
|
|
\item[$\bullet~~$]C désigne l'évènement : \og le pot est conforme \fg.
|
|
\end{itemize}
|
|
\setlength\parindent{0mm}
|
|
|
|
Pour tout évènement $E$, on note $p(E)$ sa probabilité et $\overline{E}$ l'évènement contraire de $E$.
|
|
|
|
\medskip
|
|
|
|
\begin{parts}
|
|
\part Compléter l'arbre de probabilités fourni en annexe.
|
|
\part
|
|
\begin{subparts}
|
|
\subpart Décrire en une phrase l'évènement $M_1 \cap \overline{C}$.
|
|
\begin{solution}
|
|
Cela signifie: "le pot provient de la machine $m_1$ et qu'il n'est pas conforme.
|
|
\end{solution}
|
|
\subpart Calculer la probabilité $p\left(M_{1} \cap \overline{C}\right)$.
|
|
\begin{solution}
|
|
Cette probabilité correspond à la feuille en vert \TODO{colorier la fleuille} donc
|
|
\begin{eqnarray*}
|
|
p\left(M_{1} \cap \overline{C}\right) & = & \frac{60}{100} \times \frac{7}{100} = 0,042
|
|
\end{eqnarray*}
|
|
4,2\% des pots produits sont produits par la machine 1 et ne sont pas conformes.
|
|
\end{solution}
|
|
\subpart Vérifier que $p\left(M_{2} \cap \overline{C}\right) = 0,008$.
|
|
\begin{solution}
|
|
Cette probabilité correspond à la feuille en bleu \TODO{colorier la fleuille} donc
|
|
\begin{eqnarray*}
|
|
p\left(M_{2} \cap \overline{C}\right) & = & \frac{40}{100} \times \frac{2}{100} = 0,008
|
|
\end{eqnarray*}
|
|
0,8\% des pots produits sont produits par la machine 2 et ne sont pas conformes.
|
|
\end{solution}
|
|
\end{subparts}
|
|
\part Justifier que $p\left(\overline{C}\right) = 0,05$. interpréter cette probabilité.
|
|
\begin{solution}
|
|
On cherche à calculer $p(\overline{C})$ ce qui correspond à la probabilité qu'un pot ne soit pas conforme cette probabilité correspond à toutes les feuilles $\overline{C}$ dans l'arbre donc
|
|
\begin{eqnarray*}
|
|
p(\overline{C}) & = & p(M_1 \cap \overline{C}) + p(M_2 \cap \overline{C}) = 0,042 + 0,008 = 0,05
|
|
\end{eqnarray*}
|
|
|
|
5\% des pots ne sont pas conformes.
|
|
|
|
\end{solution}
|
|
\part On prélève au hasard un pot parmi les pots non-conformes.
|
|
|
|
Déterminer la probabilité qu'il provienne de la machine $m_{2}$.
|
|
\begin{solution}
|
|
On veut calculer la probabilité d'avoir un pot venant de la machine $m_2$ parmi les pots non-conformes. On calcule donc
|
|
\begin{eqnarray*}
|
|
P_{\overline{C}}(M_2) & = & \frac{p(\overline{C} \cap M_2)}{p(\overline{C})} = \frac{0,008}{0,05} = 0,16
|
|
\end{eqnarray*}
|
|
|
|
|
|
\end{solution}
|
|
|
|
\begin{EnvUplevel}
|
|
\bigskip
|
|
|
|
\textbf{Partie B}
|
|
|
|
\medskip
|
|
|
|
L' entreprise SAPIQ reçoit un agent commercial vantant les mérites d'une nouvelle machine. La masse de moutarde contenue dans un pot produit par cette nouvelle machine est modélisée par une variable aléatoire $X$. On admet que $X$ suit une loi normale de moyenne $800$ et d'écart type $6$.
|
|
|
|
\medskip
|
|
\end{EnvUplevel}
|
|
|
|
\part
|
|
\begin{subparts}
|
|
\subpart Calculer $P(X > 810)$.
|
|
\begin{solution}
|
|
Comme $X$ suit une loi normale de moyenne $800$ et d'écart-type 6, pour calculer $P(X>810)$ on tape sur sa calculatrice:
|
|
\begin{center}
|
|
\verb+normalFRep(810, 10^99, 800,6)+
|
|
\end{center}
|
|
Et on obtient $P(X > 810) = 0,048$
|
|
\end{solution}
|
|
\subpart Calculer la probabilité arrondie au millième, qu'un pot produit par la nouvelle machine soit conforme.
|
|
\begin{solution}
|
|
Pour qu'un pot soit conforme, il faut que sa masse comprise entre 790 et 810g.
|
|
\begin{eqnarray*}
|
|
P(790 < X < 810) & = & 0,904
|
|
\end{eqnarray*}
|
|
\end{solution}
|
|
\end{subparts}
|
|
|
|
\part
|
|
\begin{subparts}
|
|
\subpart Déterminer l'intervalle centré 800 et contenant 95\% des masses des pots.
|
|
\begin{solution}
|
|
L'intervalle contenant 95\% des valeurs et centré en 800 est $\intFF{\mu -2\sigma}{\mu + 2\sigma}$ donc
|
|
\begin{eqnarray*}
|
|
\intFF{800 - 2 \times 6}{800 + 2 \times 6} & \mbox{ donc } & \intFF{788}{812}
|
|
\end{eqnarray*}
|
|
|
|
\end{solution}
|
|
\subpart L'agent commercial avance l'argument suivant : \og $X$ suit une loi normale de moyenne $800$ et d'écart type $6$. Cela signifie que tous les pots produits par notre machine contiennent entre $794$ et $806$~g de moutarde ; ils sont donc tous conformes. \fg
|
|
|
|
L'argument de l'agent commercial est-il exact ? Justifier.
|
|
\begin{solution}
|
|
D'après la question précédente, 95\% des pots produits contiennent entre 788 et 812 grammes de moutarde. Ce qui signifie que 5\% sont en dehors de cet intervalle et donc non conformes. L'argument de l'agent commercial n'est donc pas exact.
|
|
|
|
\end{solution}
|
|
\end{subparts}
|
|
|
|
\end{parts}
|
|
|
|
|
|
\vfill
|
|
|
|
\clearpage
|
|
\question[6]
|
|
On construit le tableau ci-dessous des indices de la féquentation des campings 4 étoiles ou plus, en prenant comme indice de référence 10à en 2004.
|
|
|
|
\begin{tabular}{|c|*{7}{c|}}
|
|
\hline
|
|
Année & 2004 & 2005 & 2006 & 2007 & 2008 & 2009 & 2010 \\
|
|
\hline
|
|
Fréquentation en miliers de nuitées & 25 156 & & 28 295 & 28897 & 30 063 & 31 212 & 32 014 \\
|
|
\hline
|
|
Indice & 100 & 105,22 & 11,48 & & 119,51 & 124,07 & \\
|
|
\hline
|
|
\end{tabular}
|
|
\begin{parts}
|
|
\part Calculer la fréquentation en 2005.
|
|
\begin{solution}
|
|
Fréquentation en 2005:
|
|
\begin{eqnarray*}
|
|
\frac{25 156 \times 105,22}{100} & = & 26469
|
|
\end{eqnarray*}
|
|
|
|
\end{solution}
|
|
\part Calculer l'indice, arrondi au centième, correspondant à l'année 2007.
|
|
\begin{solution}
|
|
Indice en 2007
|
|
\begin{eqnarray*}
|
|
\frac{28897 \times 100}{25156} & = & 114,87
|
|
\end{eqnarray*}
|
|
|
|
\end{solution}
|
|
\part
|
|
\begin{subparts}
|
|
\subpart Calculer le taux d'évolution global de la fréquentation entre 2004 et 2010. On donnera le résultat en pourcentage à 0,01 près.
|
|
\begin{solution}
|
|
Taux d'évolution entre 2004 et 2010
|
|
\begin{eqnarray*}
|
|
\frac{y_2 - y_1}{y_1} & = & \frac{32014 - 25156}{25156} = 0,2726 = 27,26\%
|
|
\end{eqnarray*}
|
|
\end{solution}
|
|
\subpart Calculer le taux d'évolution annuel moyen de la fréquentation entre 2004 et 2010.On donnera le résultat en pourcentage à 0,01 près.
|
|
\begin{solution}
|
|
On note $t_m$ le taux d'évolution annuel moyen entre 2004 et 2010 (6 évolutions). On a donc
|
|
\begin{eqnarray*}
|
|
(1 + t_m)^6 & = & 1 + 0,2726 \\
|
|
1 + t_m & = & 1,2627^{1/6} \\
|
|
t_m & = & 1,04099 - 1 = 0,04099 \approx 0,0410 = 4,10\%
|
|
\end{eqnarray*}
|
|
|
|
\end{solution}
|
|
\end{subparts}
|
|
\part On suppose que la fréquentation continue à augmenter de 4,10\% par an. Quelle sera alors la fréquentation en 2015?
|
|
\begin{solution}
|
|
De 2010 à 2015, il y aura eut 5 évolutions. Donc pour calculer la fréquentation en 2015, on fait
|
|
\begin{eqnarray*}
|
|
32 014 \times (1 + \frac{4,1}{100})^5 & = & 39137
|
|
\end{eqnarray*}
|
|
La fréquentation sera de 39137 nuités en 2015.
|
|
\end{solution}
|
|
\end{parts}
|
|
|
|
|
|
|
|
\vfill
|
|
\question[4]
|
|
Après réalisation d'une enquête, on estime que le temps en minutes, consacré quotidiennement par un élève à faire ses devoirs scolaires, est une variable aléatoire $X$ suivant une loi normale, d'espérance $60$ et d'écart type~$15$.
|
|
|
|
L'allure de la courbe de densité de cette loi normale est représentée ci-dessous.
|
|
|
|
L'égalité $P(X \leqslant 40) = 0,0912$ est illustrée graphiquement.
|
|
|
|
\begin{center}
|
|
\includegraphics[scale=0.3]{./fig/gaussienne}
|
|
\end{center}
|
|
|
|
\begin{parts}
|
|
\part La probabilité qu'un élève consacre quotidiennement plus de 40min à faire ses devoirs scolaires est:
|
|
\begin{oneparchoices}
|
|
\choice 0,0912
|
|
\choice 0,8076
|
|
\choice 0,8
|
|
\choice 0,9088
|
|
\end{oneparchoices}
|
|
|
|
\part La probabilité qu'un élève consacre quotidiennement plus de $80$~minutes à faire ses devoirs scolaires est:
|
|
|
|
\begin{oneparchoices}
|
|
\choice 0,0912
|
|
\choice 0,8076
|
|
\choice 0,8
|
|
\choice 0,9088
|
|
\end{oneparchoices}
|
|
|
|
\part La probabilité qu'un élève consacre quotidiennement entre 40 et 80 minutes à faire ses devoirs scolaires est:
|
|
|
|
\begin{oneparchoices}
|
|
\choice 0,817
|
|
\choice 0,95
|
|
\choice 0,5
|
|
\choice 0
|
|
\end{oneparchoices}
|
|
|
|
\part La probabilité qu'un élève consacre quotidiennement moins d'une heure à faire ses devoirs scolaires est :
|
|
|
|
\begin{oneparchoices}
|
|
\choice 0,5
|
|
\choice 0,6
|
|
\choice 1
|
|
\choice 0,1368
|
|
\end{oneparchoices}
|
|
|
|
|
|
\end{parts}
|
|
|
|
|
|
\vfill
|
|
\end{questions}
|
|
|
|
\clearpage
|
|
|
|
\Large
|
|
\begin{center}
|
|
Annexe \\ À rendre avec le copie \\ Exercice 1
|
|
|
|
\hfill
|
|
|
|
\begin{tikzpicture}[scale=1.4]
|
|
\node (root) at (0,0) {$\bullet$};
|
|
\node (Mu) at (-3, -2) {$M_1$};
|
|
\node (MuNC) at (-4, -5) {$\overline{C}$};
|
|
\node (MuC) at (-1, -5) {$C$};
|
|
\node (Md) at (3, -2) {$M_2$};
|
|
\node (MdNC) at (1, -5) {$\overline{C}$};
|
|
\node (MdC) at (4, -5) {$C$};
|
|
\draw[->] (root) -- (Mu) node[midway, left] {...};
|
|
\draw[->] (Mu) -- (MuNC) node[midway, left] {...};
|
|
\draw[->] (Mu) -- (MuC) node[midway, right] {...};
|
|
\draw[->] (root) -- (Md) node[midway, right] {...};
|
|
\draw[->] (Md) -- (MdNC) node[midway, left] {...};
|
|
\draw[->] (Md) -- (MdC) node[midway, right] {...};
|
|
\end{tikzpicture}
|
|
\end{center}
|
|
|
|
\hfill
|
|
|
|
\end{document}
|
|
|
|
%%% Local Variables:
|
|
%%% mode: latex
|
|
%%% TeX-master: "master"
|
|
%%% End:
|
|
|